***
This Logic Games question is from the June 2004 LSAT.
We don't see an obvious solution based on the rules, so let's diagram and try to find a situation where the scenario in the question is true, so we can answer the question. We'll take as a starting point the correct diagram we had on the last question and alter it:
X: T I F
Y: F H S
Z: T I P
We want to make H serve more buildings than I. How many will that have to be? Well, the rules say I serves more than S, so I must serve at least two. For H to serve more than I, H must serve all three. So, let's add H to X and Z to make that true:
X: T I F H
Y: F H S
Z: T I P H
We see this conforms to all the rules and was much faster than creating a diagram from scratch. It's better to alter and redraw an old one. We can see the only valid one in this correct diagram is E, so E is correct.
Remember:
Use a diagram to solve the problem if you can't deduce anything directly from the rules, and alter a diagram from a previous question (rather than making a brand new one) whenever possible, to save time.
Wouldn't an easier solution simply to be to look at the last rule that says the taco truck serves two of the same buildings as the ice cream truck?
ReplyDeleteBased on your final diagram, the falafel truck serves Y with the hot dog truck and the falafel truck and the salad truck also serve Y. The ice cream and pita truck also both serve Z.All three of these are answer choices in addition to E.
I'm confused.
I think I get it know. The question stem asks for "buildings" not "building" which means more than one.
ReplyDeleteNow I see why E is correct.
Yeah, the idea is which ones serve the exact same buildings. The correct diagram eliminated all choices but E (since they didn't appear in that diagram, and if they had to be true, they would've appeared).
ReplyDelete